Vanilla Ice cream comes in 2 sizes. The 48 oz box costs $1.97. The 128 oz costs $5.87. Which size is the better buy?

Answers

Answer 1

Answer: The 128 oz box is the better buy.


Related Questions

line
A storage bin has the shape of a cylinder with a conical top. What is the volume of the storage bin if
its radius is r = 4.9 ft, the height of the cylindrical portion is h = 9.7 ft, and the overall height is
H = 16.3 ft?
Volume (to the nearest tenth)

Answers

Answer:

Step-by-step explanation:

To find the volume of the storage bin, we need to calculate the volumes of both the cylindrical portion and the conical top, and then add them together.

The volume of the cylindrical portion can be calculated using the formula:

V_cylinder = π * r^2 * h

where r is the radius and h is the height of the cylindrical portion.

Substituting the given values, we have:

V_cylinder = π * (4.9 ft)^2 * 9.7 ftV_cylinder ≈ 748.07 ft³ (rounded to two decimal places)

The volume of the conical top can be calculated using the formula:

V_cone = (1/3) * π * r^2 * H_cone

where r is the radius and H_cone is the height of the conical top.

The height of the conical top can be obtained by subtracting the height of the cylindrical portion from the overall height:

H_cone = H - h = 16.3 ft - 9.7 ft = 6.6 ft

Substituting the given values, we have:

V_cone = (1/3) * π * (4.9 ft)^2 * 6.6 ftV_cone ≈ 243.24 ft³ (rounded to two decimal places)

To find the total volume, we add the volume of the cylindrical portion and the volume of the conical top:

Total volume = V_cylinder + V_cone

Total volume ≈ 748.07 ft³ + 243.24 ft³

Total volume ≈ 991.31 ft³ (rounded to one decimal place)

Therefore, the volume of the storage bin is approximately 991.3 ft³ (rounded to the nearest tenth).

Thus the required volume is, 975.05  ft³

Given that,

radius = r = 4.9

Height of cylindrical potion = h = 9.7

Overall height = 16.3

Since,

total height = Height of the cylinder + height of the cone

Height of the cone = 16.3 - 9.7

                                = 6.6 m

Since we know that,

Volume of a cylinder = πr² h

⇒ π (4.9)²(9.7)

⇒ 731.29 ft³

Since we also know that

Volume of a cone = (1/3)πr² h

= 731.29/3

= 243.76  ft³

Volume of the bin = volume of cone + volume of cylinder

= 731.29 ft³  + 243.76  ft³

Hence the volume be,

= 975.05  ft³

To learn more about volume visit:

https://brainly.com/question/16860802

#SPJ1

Which is the equation of the given line in point-slope form?

y−0=−1(x−8)

y−0=1(x+8)

y=−x+8

y−8=−1(x+0)

Answers

Answer:

y = -x + 8

Step-by-step explanation:

Let's break down the equation step by step to understand it better.

The equation in point-slope form is given as:

y - y1 = m(x - x1)

In this case, we have:

y - 0 = -1(x - 8)

The point-slope form uses a specific point (x1, y1) on the line and the slope (m) of the line.

Here, the point (x1, y1) is (8, 0), which represents a point on the line. This means that when x = 8, y = 0. The graph has a point at (8, 0), which confirms this information.

The slope (m) is -1 in this equation. The slope represents the rate at which y changes with respect to x. In this case, since the slope is -1, it means that for every unit increase in x, y decreases by 1. The negative sign indicates that the line has a downward slope.

By substituting the values into the equation, we get:

y - 0 = -1(x - 8)

Simplifying further:

y = -x + 8

This is the final equation of the line in slope-intercept form. It tells us that y is equal to -x plus 8. In other words, the line decreases by 1 unit in the y-direction for every 1 unit increase in the x-direction, and it intersects the y-axis at the point (0, 8).

If the graph has points at (0, 8) and (8, 0), the equation y = -x + 8 accurately represents that line.

Two homebuyers are financing $137,000 to purchase a condominium. They obtained a 15-year, fixed-rate loan with a rate of 5.05%. They have been given the option of purchasing up to four points to lower their rate to 4.81%. How much will the four points cost them?

$1,370
$1,730
$4,580
$5,480

Answers

The cost of four points is:4 x $1,370 = $5,480Thus, the four points will cost the homebuyers $5,480.

Points can help lower mortgage rates on fixed-rate loans. The concept of points, which are basically prepaid interest, is a little complicated.

Each point is worth one percent of the loan amount, and paying points can lower your interest rate by a certain amount, typically about one-eighth to one-quarter of a percentage point.

The cost of points in the given scenario can be found using the following steps:

The loan amount to purchase a condominium is $137,000. The homebuyers obtained a 15-year fixed-rate loan with a rate of 5.05%.

If the homebuyers opt for four points, their loan rate will decrease to 4.81%.

To figure out how much the points will cost the homebuyers, we must first determine the cost of one point. Since one point is equal to 1% of the loan amount, one point on a $137,000 loan is:1% of $137,000 = $1,370

To learn more about : cost

https://brainly.com/question/2292799

#SPJ8

Edwin sells jars of jam for $1.90 each. Determine how many jars of jam Edwin needs to sell to break even if the variable cost per jar is $1.10 and fixed expenses are $35,700.00 per year.

Answers

Edwin needs to sell 44,625 jars of jam to break even.

To determine how many jars of jam Edwin needs to sell to break even, we'll calculate the breakeven point using the following formula:

Breakeven Point = Fixed Expenses / (Selling Price per Unit - Variable Cost per Unit)

Given information:

Selling Price per Unit (SP) = $1.90

Variable Cost per Unit (VC) = $1.10

Fixed Expenses = $35,700.00 per year

Plugging in the values into the formula:

Breakeven Point = $35,700 / ($1.90 - $1.10)

Breakeven Point = $35,700 / $0.80

Breakeven Point = 44,625 jars

Therefore, Edwin needs to sell 44,625 jars of jam to break even.

for such more question on breakeven point

https://brainly.com/question/30551452

#SPJ8

I NEED HELP WITH STATISTICS

Answers

(a) The null hypothesis is that the mean birth weight of babies born at full term is 7.2 pounds. The alternative hypothesis is that the mean birth weight of babies born at full term is greater than 7.2 pounds.

(b) If the scientist decides to reject the null hypothesis, she might be making a Type I error.

(c) A Type II error occurs when the null hypothesis is false, but the scientist fails to reject it.

How to explain the information

a A Type I error occurs when the null hypothesis is true, but the scientist rejects it. In this case, the null hypothesis is that the mean birth weight of babies born at full term is 7.2 pounds. If the scientist rejects this hypothesis, she is saying that she believes that the mean birth weight is greater than 7.2 pounds. However, if the null hypothesis is true, then the mean birth weight is actually 7.2 pounds, and the scientist has made a mistake.

b In this case, the scientist would fail to reject the null hypothesis and conclude that the mean birth weight of babies born at full term is 7.2 pounds. However, the true mean birth weight is 7.7 pounds, so the scientist would be making a Type II error.

c In the context of a Type II error, suppose the null hypothesis is false, meaning there is indeed a significant difference or relationship. However, due to various factors such as insufficient sample size, low statistical power, or other limitations, the scientist fails to reject the null hypothesis. Consequently, they accept the null hypothesis even though it is false, leading to a Type II error.

Learn more about hypothesis on

https://brainly.com/question/606806

#SPJ1

You spin the spinner once. 123 What is P(less than 2)? Write your answer as a fraction or whole number.

Answers

Answer:

See below

Step-by-step explanation:

Since the spinner has the numbers 1, 2, and 3 on it, and we want to find the probability of spinning a number less than 2, there is only one possible outcome that satisfies this condition, which is spinning a 1. Therefore, the probability of spinning a number less than 2 is:

P(less than 2) = P(1) = 1/3

So the probability of spinning a number less than 2 is 1/3.

Express 75 as a product of its prime factors write the prime factors in ascending order and give your answer in index form

Answers

Step-by-step explanation:

75 = 3 x 5 x 5    in prime factorization

Answer:

Step-by-step explanation:

3x5x5

what is the greatest common factor of 97 and 24? what the answer

Answers

1

Because the number 97 is a prime number

Answer:

The greatest common factor (GCF) of two numbers is the largest number that divides evenly into both numbers. Since 97 is a prime number and 24 is not divisible by 97, the GCF of 97 and 24 is 1.

please help! mathematicians

Answers

Answer:

1 < m < 4

Step-by-step explanation:

If the roots of function f(x) are not real, then the discriminant (the part under the square root sign) will be negative.

Set the discriminant less than zero and rewrite in standard form:

[tex]\begin{aligned}16-4m(-m+5)& < 0\\16+4m^2-20m& < 0\\4m^2-20m+16& < 0\\4(m^2-5m+4)& < 0\\m^2-5m+4& < 0\end{aligned}[/tex]

Factor the quadratic:

[tex]\begin{aligned}m^2-5m+4& < 0\\m^2-4m-m+4& < 0\\m(m-4)-1(m-4)& < 0\\(m-1)(m-4)& < 0\end{aligned}[/tex]

The leading coefficient of the quadratic m² - 5m + 4 is positive.

Therefore, the graph will be a parabola that opens upwards.

This means that the interval where the parabola is below the x-axis (negative) is between the zeros of the quadratic. Since the zeros are m = 1 and m = 4, the solution to the inequality is 1 < m < 4.

Therefore, the values of m for which the roots of function f(x) will be non-real are 1 < m < 4.

3) Last year the mean salary for professors in a particular community college was $62,000 with a standard deviation of $2000. A new two year contract is negotiated. In the first year of the contract, each professor receives a $1500 raise.

Find the mean and standard deviation for the first year of the contract.
b) In the second year of the contract, each professor receives a 3% raise based on their salary during the first year of the contract. Find the mean and the standard deviation for the second year of the contract.

Answers

a) Mean for the first year of the contract: $63,500

The standard deviation for the first year of the contract: $2,000.

b) Mean for the second year of the contract: $65,405.

The standard deviation for the second year of the contract: $60.

We have,

To find the mean and standard deviation for the first year of the contract, we can use the given information and the properties of the normal distribution.

Given:

The mean salary for professors in the previous year = $62,000

Standard deviation in the previous year = $2,000

Raise in the first year = $1,500

Mean for the first year of the contract:

The mean salary for the first year can be obtained by adding the raise to the previous mean:

Mean = Previous Mean + Raise

Mean = $62,000 + $1,500

Mean = $63,500

The standard deviation for the first year of the contract:

Since each professor receives the same raise, the standard deviation remains the same:

Standard Deviation = $2,000

Therefore, for the first year of the contract, the mean salary is $63,500, and the standard deviation remains $2,000.

Now,

In the second year of the contract, each professor receives a 3% raise based on their salary during the first year of the contract.

To find the mean and standard deviation for the second year, we can use the given information and the properties of the normal distribution.

Mean for the second year of the contract:

To calculate the mean for the second year, we need to add a 3% raise to the mean salary of the first year:

Mean = Mean of the first year + (3% * Mean of the first year)

Mean = $63,500 + (0.03 * $63,500)

Mean = $63,500 + $1,905

Mean = $65,405

The standard deviation for the second year of the contract:

Since each professor receives a raise based on their salary from the first year, the standard deviation also increases. To calculate the standard deviation, we multiply the standard deviation from the first year by the percentage increase:

Standard Deviation = Standard Deviation of the first year * (Percentage Increase / 100)

Standard Deviation = $2,000 * (3 / 100)

Standard Deviation = $2,000 * 0.03

Standard Deviation = $60

Therefore, for the second year of the contract, the mean salary is $65,405, and the standard deviation is $60.

Thus,

a) Mean for the first year of the contract: $63,500

The standard deviation for the first year of the contract: $2,000.

b) Mean for the second year of the contract: $65,405.

The standard deviation for the second year of the contract: $60.

Learn more about mean here:

https://brainly.com/question/23263573

#SPJ1

The amount of time a certain brand of light bulb lasts is normally distributed with a mean of 2000 hours and a standard deviation of 25 hours. Out of 665 freshly installed light bulbs in a new large building, how many would be expected to last between 2030 hours and 2060 hours, to the nearest whole number?

Answers

To determine the number of light bulbs expected to last between 2030 hours and 2060 hours, we need to calculate the z-scores corresponding to these values and then use the z-score formula to find the proportion of light bulbs within this range.

The z-score formula is given by:

z = (x - μ) / σ

where:

x = value

μ = mean

σ = standard deviation

For 2030 hours:

z1 = (2030 - 2000) / 25

For 2060 hours:

z2 = (2060 - 2000) / 25

Now, we can use the z-scores to find the proportions associated with each value using a standard normal distribution table or calculator. The table or calculator will provide the area/proportion under the normal curve between the mean and each z-score.

Let's calculate the z-scores and find the proportions:

z1 = (2030 - 2000) / 25 = 1.2

z2 = (2060 - 2000) / 25 = 2.4

Using a standard normal distribution table or calculator, we can find the proportions corresponding to these z-scores:

P(z < 1.2) ≈ 0.8849

P(z < 2.4) ≈ 0.9918

To find the proportion of light bulbs expected to last between 2030 hours and 2060 hours, we subtract the cumulative probabilities:

P(2030 < x < 2060) = P(z1 < z < z2) = P(z < z2) - P(z < z1)

P(2030 < x < 2060) ≈ 0.9918 - 0.8849

Finally, we multiply this proportion by the total number of light bulbs (665) to get the estimated number of light bulbs expected to last between 2030 hours and 2060 hours:

Number of light bulbs ≈ (0.9918 - 0.8849) * 665

Rounding to the nearest whole number, the expected number of light bulbs that would last between 2030 hours and 2060 hours is approximately 71.[tex]\huge{\mathfrak{\colorbox{black}{\textcolor{lime}{I\:hope\:this\:helps\:!\:\:}}}}[/tex]

♥️ [tex]\large{\textcolor{red}{\underline{\mathcal{SUMIT\:\:ROY\:\:(:\:\:}}}}[/tex]

A scientist mixes water (containing no salt) with a solution that contains 35% salt. She wants to obtain 140 ounces of a mixture that is 15% salt. How many
ounces of water and how many ounces of the 35% salt solution should she use?

Answers

Answer:

.35x = 140(.15)

.35x = 21

x = 60 oz of 35% salt.

The scientist will need 60 oz of the 35% salt solution and 80 oz of water.

How do you solve the question Deloitte signs a contract on December 1 to provide 40 days of advisory services with receipt of $20,000 due at the end of the contract. On December 31, 75% of the services have been completed.

Answers

As of December 31, Deloitte should recognize $15,000 as revenue for the advisory services completed.

To solve the given question, we need to determine the amount of revenue that Deloitte should recognize as of December 31, based on the percentage of services completed.

Here's how we can calculate it:

Calculate the total revenue for the contract:

Total revenue = $20,000

Determine the percentage of services completed:

Percentage of services completed = 75%

Calculate the revenue recognized as of December 31:

Revenue recognized = Percentage of services completed × Total revenue

= 75% × $20,000

= $15,000

Therefore, as of December 31, Deloitte should recognize $15,000 as revenue for the advisory services completed.

Learn more about revenue click;

https://brainly.com/question/29567732

#SPJ1

Suppose there are 17 jelly beans in a box-2 red, 3 blue, 4 white, and 8 green. What part of the jelly beans is blue? As a decimal rounded to the nearest ten-thousandth (four decimal places)

Answers

Blue Jelly beans are 0.1764 part of total .

Given,

Total beans = 17

Blue = 3

Red =2

White =4

Green =8

Now,

Out of total , green jelly beans = 8/17

Out of total , red jelly beans = 2/17

Out of total , white jelly beans = 4/17

Out of total , blue jelly beans = 3/17

Hence the blue jelly beans are 0.1764 part of total jelly beans .

Know more about decimal,

https://brainly.com/question/8985071

#SPJ1

QUESTION 1 1.1 1.2 1.4 Use the definition of the derivative (first principles) to determine f'(x) if f(x)=2x 1.3 Determine f'(x) from first principles if f(x)=9-x². Determine f'(x) from first principles if f(x)=-4x².​

Answers

Based on the functions given, it should be noted that the values will be 2, -2x and -8x.

How to calculate the value

Using the definition of the derivative, we have:

f'(x) = lim(h->0) [f(x + h) - f(x)] / h

= lim(h->0) [2(x + h) - 2x] / h

= lim(h->0) 2h / h

= lim(h->0) 2

= 2

Therefore, f'(x) = 2.

For f(x) = 9 - x²:

Using the definition of the derivative, we have:

f'(x) = lim(h->0) [f(x + h) - f(x)] / h

= lim(h->0) [9 - (x + h)² - (9 - x²)] / h

= lim(h->0) [9 - (x² + 2xh + h²) - 9 + x²] / h

= lim(h->0) [-2xh - h²] / h

= lim(h->0) (-2x - h)

= -2x

Therefore, f'(x) = -2x.

For f(x) = -4x²:

Using the definition of the derivative, we have:

f'(x) = lim(h->0) [f(x + h) - f(x)] / h

= lim(h->0) [-4(x + h)² - (-4x²)] / h

= lim(h->0) [-4(x² + 2xh + h²) + 4x²] / h

= lim(h->0) [-4x² - 8xh - 4h² + 4x²] / h

= lim(h->0) [-8xh - 4h²] / h

= lim(h->0) (-8x - 4h)

= -8x

Therefore, f'(x) = -8x.

Learn more about functions on

https://brainly.com/question/31878183

#SPJ1

Determine the a) total annual cost, and b) cost per mile to the nearest cent.
1. Liz Nolan drove 34,500 miles last year. The total of fixed costs was $9,916 and of variable costs was
$4,897.

Answers

Answer:

total annual cost: 49313

cost per mile: 14 cents

Step-by-step explanation:

find total annual cost by adding everything up

find cost per mile by doing 4897/34500

cost/ miles

we use variable cost since the only thing that might change each year is the amount of miles they drive

fixed costs are fixed and don't change

Find the measure of ∠F
.

Answers

Step-by-step explanation:

triangle EFG is an isosceles triangle

angle G

= 180°-58°

= 122° (adj. angles on a str. line)

angle F

= (180°-122°)÷2

= 29° (angles in a triangle)

(q11) Find the center of mass of the system of objects that have masses 2 , 3 and 5 at the point (-1,2),(1,1) and (3,3) respectively.

Answers

The center of mass of the system is approximately (3.7, 2.6).

The center of mass of a system of objects is the point where all the weight of the system appears to be concentrated. It can be defined as the average location of the weighted parts of the system.

The center of mass of a system is dependent on the mass of the objects in the system and their positions.

Let's determine the center of mass of the system with masses of 2, 3, and 5 at the points (-1, 2), (1, 1), and (3, 3), respectively. Let's name the masses m1, m2, and m3, respectively, and the coordinates (x1, y1), (x2, y2), and (x3, y3).

The x-component of the center of mass is given by the formula:

x= (m1x1 + m2x2 + m3x3) / (m1 + m2 + m3)

The y-component of the center of mass is given by the formula:

y= (m1y1 + m2y2 + m3y3) / (m1 + m2 + m3)

By using the given values, let's calculate the x and y components of the center of mass:

x = (2 x -1 + 3 x 1 + 5 x 3) / (2 + 3 + 5) = 37/10 ≈ 3.7y

= (2 x 2 + 3 x 1 + 5 x 3) / (2 + 3 + 5)

= 26/10 = 2.6

To learn more about : mass

https://brainly.com/question/28916233

#SPJ8

546, 400 and 4,856 The value of 4 in which number is how many times larger than the value of 4 in which number.​

Answers

To determine how many times larger the value of 4 is in the second number compared to the first number, we need to calculate the ratio of the values.

First number: 546
Second number: 4,856

In the first number, the value of 4 is the same as the digit itself since it appears once.

In the second number, the value of 4 is larger since it appears twice.

To find the ratio, we divide the value of 4 in the second number by the value of 4 in the first number:

Value of 4 in second number: 2
Value of 4 in first number: 1

Ratio: 2/1 = 2

Therefore, the value of 4 in the second number is two times larger than the value of 4 in the first number.

'Two containers designed to hold water are side by side, both in the shape of a cylinder. Container A has a diameter of 12 feet and a height of 14 feet. Container B has a diameter of 10 feet and a height of 20 feet. Container A is full of water and the water is pumped into Container B until Container B is completely full.To the nearest tenth, what is the percent of Container A that is full after the pumping

Answers

The nearest tenth, approximately 93.5% of Container A is full after the water is pumped into Container B.

To determine the percentage of Container A that is full after the water is pumped into Container B, we need to compare the volumes of the two containers.

The volume of a cylinder can be calculated using the formula: V = πr^2h, where V is the volume, π is a constant (approximately 3.14159), r is the radius, and h is the height.

For Container A:

Radius (r) = Diameter / 2 = 12 ft / 2 = 6 ft

Height (h) = 14 ft

For Container B:

Radius (r) = Diameter / 2 = 10 ft / 2 = 5 ft

Height (h) = 20 ft

Now, let's calculate the volumes of the two containers:

Volume of Container A = π * (6 ft)^2 * 14 ft ≈ 1,679.65 ft^3

Volume of Container B = π * (5 ft)^2 * 20 ft ≈ 1,570.8 ft^3

To find the percentage of Container A that is full, we need to calculate the ratio of the volume of water in Container B to the volume of Container A:

Ratio = Volume of Container B / Volume of Container A

Ratio = 1,570.8 ft^3 / 1,679.65 ft^3 ≈ 0.9347

Finally, to convert this ratio to a percentage, we multiply it by 100:

Percentage = Ratio * 100

Percentage ≈ 0.9347 * 100 ≈ 93.5%

Therefore, to the nearest tenth, approximately 93.5% of Container A is full after the water is pumped into Container B.

for such more question on percentage

https://brainly.com/question/24877689

#SPJ8


Minka pours 1/4 cup of milk on her oatmeal each day for 7

Answers

Assuming you want the amount of milk in 7 days, we can set up a multiplication problem. Given Minka pours 1/4 cup of milk in her oatmeal each day, we can multiply that by 7 days to find that:

1/4 = 0.25
0.25 • 7 = 1.75, or 1 3/4

By the end of 7 days, Minka pours 1 3/4 cups of milk into her oatmeal collectively.

22% of what number is 3300

Answers

To find the number that corresponds to 22% of a given value, you can divide the given value by 22% (or 0.22).

Let's use this approach to find the number:

3300 ÷ 0.22 = 15,000

So, 22% of 15,000 is equal to 3300.

Answer:

x = 15000

Step-by-step explanation:

If you are using a calculator, simply enter 3300×100÷22, which will give you the answer.

6 I need steps to know how we did it

Answers

Answer:

D

Step-by-step explanation:

the right triangle contains h , the horizontal leg and the sloping side which is the hypotenuse of the right triangle.

the horizontal leg is half the measure of the side of the square base.

horizontal leg = 8 ÷ 2 = 4

using Pythagoras' identity in the right triangle

the square on the hypotenuse is equal to the sum of the squares on the other 2 sides, that is

h² + 4² = 10² ( subtract 4² from both sides )

h² = 10² - 4² ( take square root of both sides )

h = [tex]\sqrt{10^2-4^2}[/tex]

Problem
Find the equation of the line.
Use exact numbers.

Answers

The Equation of line is y= -3/2x + 60

From the graph we take two coordinates as (2, 0) and (0, 3)

We know the formula for slope

Slope= (Change in y)/ (Change in x)

Slope = (3-0)/ (0-2)

Slope= 3 / (-2)

Slope= -3/2

Now, Equation of line

y - 0 = -3/2 (x-  2)

y= -3/2x + 6

Thus, the Equation of line is y= -3/2x + 60.

Learn more about Slope here:

https://brainly.com/question/3605446

#SPJ1

The number of combinations of eight items taken three at a time can be written as

Answers

Answer: 8C3

Step-by-step explanation: You need to use Combinations for this. Out of 8, you need to select 3, so answer is 8C3.

Multiply three consecutive digits backwards starting from 8, and divide by 3 factorial

(8*7*6)/(3*2*1)

=56

Jessica needs to know how much water her new fish tank can hold:

A rectangular prism with a length of 8 inches, a width of 4 inches, and a height of 9 inches.

Determine the total volume of the fish tank.

Answers

The fish tank has a total volume of 288 inch³. As a result, Jessica's new fish tank has a capacity of 288 inch³ for water.

The volume of a rectangular prism can be calculated using the formula:

V = l x b x h..........(i)

where,

V ⇒ Volume

l  ⇒ length

b ⇒ width

h ⇒ height

From the question, we are given the values,

l = 8 inches

b = 4 inches

h = 9 inches

Putting these values in equation (i), we get,

V = 8 x 4 x 9

⇒ V = 288 in³

Therefore, the fish tank has a total volume of 288 inch³. As a result, Jessica's new fish tank has a capacity of 288 inch³ for water.

Learn more about the volume of rectangular prism on:

https://brainly.com/question/24284033

You purchase a tarp to cover the driveway when it snows. The
dimensions of your driveway are 10.2 ft. by 15.7 ft. If the tarp covers
your entire driveway, how many square feet are covered? Your answer
should be a number only. Do not round.

Answers

If the dimensions of your driveway are 10.2 ft. by 15.7 ft and the tarp covers your entire driveway,  the square feet are covered is [tex]160.14ft^{2}[/tex]

How can the dimension be calculated?

In mathematics, a dimension is the length or width of an area, region, or space in one direction. It is just the measurement of an object's length, width, and height.

With the given conditions,  we can formulate the expression as

;10.2 ft. * 15.7 ft

=160.14

Learn more about dimensions at;

https://brainly.com/question/26740257

#SPJ1

Round to the nearest given place.
1.45169 thousandths

Answers

Answer:

1.452

Step-by-step explanation:

1.45169 rounded to the thousandths place would be 1.452

I need the solution!!!!​

Answers

Solve for the first variable in one of the equations, then substitute the result into the other equation.

Point form :
(-4,0)

Equation form :
x = -4, y = 0

I NEED HELP WITH STATISTICS

Answers

Am here for you so need anything don’t message me
Other Questions
Question 3 a) Why do nations engage in international business? That is, what are the benefits of international trade and investment? b) How can firms show corporate social responsibility in emerging markets and developing economies? Question 4 a) What are the specific characteristics of democracy? How do these characteristics facilitate international business? b) What is the role of FDI, licensing and joint ventures in reducing the impact of import tariffs? KPI and the corresponding tool(s) that can be adopted by Baros to monitor the productivity, efficiency, quality and flexibility of its business. Explain how each index is applied in Baros. The _________ of an ad serves to inform, persuade, and stimulate buying action.A. body copy B. headline C. illustration D. signature On July 1, 2021, Timmy Corp. purchased a building for $500,000. Depreciation estimated at $5,000 for the year and has not been recorded for 2021. What type of adjustment is needed at the fiscal year end December 31, 2021, and why?Accrued expense. The company has used the building during the year and a portion of its historical cost needs to be allocated to expenseDeferred expense. The building has been used during the year and a portion of its historical cost is recorded as an expenseDeferred expense. The expense of using the building has not been recorded and the payment of depreciation expense is deferred to the next period.Depreciation is only an estimate. No adjustment is needed.Accrued expense. The purchase of the building needs to be recorded Suppose the inverse demand curve on ore is given by P = 77 -0.15 Q. Ore can be either mined or obtained through a recycling program. The marginal cost of mining is MC1 = 991. The marginal cost of obtaining ore through recycling is MC = 17 + 2 92. What percent of total demand is satisfied by recycled ore (express your answer in percentage, i.e., if the answer is 45.34% then enter 45.34)? ind the least-squares regression line y^=b0+b1x through thepoints(1,1),(2,6),(5,13),(9,19),(12,23)and then use it to find point estimates y^ corresponding to x=1and x=7.For x=1, y^ =For x=7, Case: you are finance director with responsibility fordesigning effective control systems.Provide all steps in reaching this goal You are exporting shell casings for small arms ammunition from the country of Wakanda to the country of Freedonia. The government of Wakanda may demand:an end-use certificate.a consular invoice.an import licence.a certificate of origin question 5 what is a minimum exam score required for promotion?themean is 62 and standard deviation is 105. Officers need to score in the top 10% on the exam in order to be considered for promotion. What is a minimum exam score required for promotion? rayne has been admitted to the hospital and is in acute alcohol withdrawal. she is given the drug , part of a category of drugs that are used to ease the physical and psychological symptoms associated with withdrawal. Zwick Company bought 24,000 shares of the voting common stock of Handy Corporation in January 2021. In December, Handy announced $204,300 net income for 2021 and declared and paid a cash dividend of $4.00 petshare on all 209,000 shares of its outstanding common stock. Zwick Company's dividend revenue from Handy Corporation in December 2021 would be: $0. $23.460. $96,000. None of the above. Assume the cross-rate trader at Deutsche Bank notices that Credit Monaco is buying dollars at S(/$)= 0.7638, the same as Deutsche Banks bid price. Similarity, he observes that Barclays is buying British pound at S ($/) =1.5400, also the same as Deutsche Bank. He next finds that Credit Agricole is making a direct market between euro and the pound, with a current ask price of S(/)=1.1705. Outline the arbitrage opportunities Jon establishes a long position of one T-bond future today for a settlement price of 101'20. The exchange requires an initial margin of $2400 and a maintenance margin of $2200. Below are the next day closing price on this contract. Day 1: settlement price101'12 The margin account balance at the end of Day 1 is____dollars A company that is about to announce the results of its clinical trials of a new product. If the trials are successful, the stock will be worth $70 per share. If the trials are unsuccessful the stock will be worth $18 per share. Suppose on that morning before the results are released the shares are trading for $55 per share. What sort of expectations do investors have about the success of the trials? kyle has already contracted with vick on a construction project. after some disagreement on materials, location, and deadlines, they decide that the original contract is no longer valid. kyle and vick now need a(n) . A drawback of brands as compared to unbranded products is that which action happens? nougat corporation wants to raise $4.1 million via a rights offering. the company currently has 470,000 shares of common stock outstanding that sell for $42 per share. its underwriter has set a subscription price of $17 per share and will charge the company a spread of 5 percent. if you currently own 5,000 shares of stock in the company and decide not to participate in the rights offering, how much money can you get by selling your rights? (do not round intermediate calculations and round your answer to 2 decimal places, e.g., 32.16.) f a company paid their property insurance 5 years in advance, where would this payment appear on the balance sheet? a 50.0 g sample of aluminum at 90.0 oc is added to a 250.0 g sample of water at 15.0 oc. what is the final temperature of the mixture? ill mark brainliest Drag the tiles to the correct boxes to complete the pairs. Not all tiles will be used.Match each graph below with the correct solution or correct number of solutions to the system of equations.#platofam